Independent learning program for GPs

Independent learning program for GPs

Unit 464Unit November 469 April 20112010

EarBipolar nose anddisorders throat

www.racgp.org.au/check Independent learning program for GPs

Independent learning program for GPs 2008–2010 triennium

The check Program is a monthly quality 2008 topics Print CD improvement and continuing professional development (QI&CPD) activity and independent • Hypertension • Patient initiated violence learning program developed by the RACGP. The • Polycystic ovary syndrome • 2008 and 2009 check Programs are available in • Evidence based medicine • Endocrinology and the skin both CD and print format. The 2010 program is • Imaging • Haematology available in print, as well as a handy, interactive • Influenza • Patient safety in general practice USB format. 2009 topics Print CD Print price per program • Male reproductive health • Type 2 diabetes • Chronic disease management • Palliative care RACGP member $161 • Chronic obstructive • Gastroenterology Nonmember $202 pulmonary disease • Motivational interviewing Medical student/registrar/AMC $86 • Fatigue • Cardiovascular emergencies Overseas nonmember $214 • Sexually transmissible infections • Sport medicine Overseas member $176 Overseas medical student/registrar/AMC $109 2010 topics Print USB • Stroke • Ophthalmology CD/USB price per program • Challenging consultations • Wound management • Anxiety disorders • Thyroid RACGP member $130 • Office based procedures • Rashes Nonmember $190 • Juvenile idiopathic arthritis and • Osteoarthritis Medical student/registrar/AMC $90 rheumatoid arthritis • Child and adolescent behaviour Overseas nonmember $175 Post, fax, phone or email your order to: Overseas member $125 The RACGP, 1 Palmerston Crescent, South Melbourne, Victoria 3205, Australia Overseas medical student/registrar/AMC $85 Fax 03 8699 0400 Tel 03 8699 0495 Email [email protected] ACN 000 223 807 ABN 34 000 223 807

Delivery details (please print clearly and retain a copy for your records)

Date Name RACGP no. / / Student/AMC no. University

Street address Suburb

State Postcode Country Telephone

Please allow up to 14 days for delivery of goods. The College must be notified of missing or incorrect orders within 28 days of placing the order. Orders returned Payment details must be received within 28 days of receipt in order to obtain refund or replacement. Prices include GST, postage and handling. Overseas prices exclude GST. Payment type

Amex Visa MasterCard Cheque (make payable to the RACGP)

Card number Expiry date Total amount / $ Cardholder name (please print clearly) Cardholder signature

FOR OFFICE USE ONLY RAGGP NO. (Bill to) ______BATCH NO. ______ORDER NO. ______INVOICE NO. ______

check 2008–2010 order form ad.indd 1 11/03/11 2:33 PM Independent learning program for GPs Independent learning program for GPs

Independent learning program for GPs 2008–2010 triennium Independent learning program for GPs The check Program is a monthly quality 2008 topics Print CD improvement and continuing professional development (QI&CPD) activity and independent • Hypertension • Patient initiated violence learning program developed by the RACGP. The • Polycystic ovary syndrome • Psychosis 2008 and 2009 check Programs are available in • Evidence based medicine • Endocrinology and the skin both CD and print format. The 2010 program is • Imaging • Haematology available in print, as well as a handy, interactive • Influenza • Patient safety in general practice USB format. 2009 topics Print CD Medical Editor Print price per program • Male reproductive health • Type 2 diabetes Catherine Dodgshun • Chronic disease management • Palliative care Editor RACGP member $161 • Chronic obstructive • Gastroenterology BipolarEar nose disorders and throat Nicole Kouros Nonmember $202 pulmonary disease • Motivational interviewing Unit 464469 NovemberApril 2011 2010 Production Coordinator Medical student/registrar/AMC $86 • Fatigue • Cardiovascular emergencies Morgan Liotta Overseas nonmember $214 • Sexually transmissible infections • Sport medicine Senior Graphic Designer Overseas member $176 From the editor 2 Jason Farrugia Overseas medical student/registrar/AMC $109 2010 topics Print USB Graphic Designer Case 1 Chris’Adriana concern has a hoarse voice 3 Beverly Jongue • Stroke • Ophthalmology Case 2 DorothyJean’s sudden is experiencing loss of hearing pain in her groin 6 Authors CD/USB price per program • Challenging consultations • Wound management June Choo • Anxiety disorders • Thyroid Case 3 Jill’sRobbie’s knee ongoing pain ear problems 9 Michael Dobson RACGP member $130 • Office based procedures • Rashes Ana Teresa Licup Case 4 AngelaNigel is presentsexperiencing with vertigopainful and nauseastiff joints 1216 Nonmember $190 • Juvenile idiopathic arthritis and • Osteoarthritis Mark Paine Medical student/registrar/AMC $90 rheumatoid arthritis • Child and adolescent behaviour Case 5 JanJohn’s has nosebleed ongoing knee pain 1621 Elizabeth Rose Philip Michael Overseas nonmember $175 Post, fax, phone or email your order to: Carmel Crock Overseas member $125 ReferencesCase 6 Claire’s onging allergies 2125 The RACGP, 1 Palmerston Crescent, South Melbourne, Victoria 3205, Australia Reviewer Fax 03 8699 0400 Tel 03 8699 0495 Email [email protected] Overseas medical student/registrar/AMC $85 ResourcesCase 7 Julie’s had a runny nose and feels congested 2228 Tracy Cheffins ACN 000 223 807 ABN 34 000 223 807 CategoryCase 8 2 QI&CPDJim has activity painful blocked ears 2431 Subscriptions Call the Subscription Coordinator Delivery details (please print clearly and retain a copy for your records) Resources and References 34 for all enquiries on 03 8699 0495 or email [email protected]. Date Name RACGP no. Category 2 QI&CPD activity 35 Published by / / The Royal Australian College of General Practitioners Student/AMC no. University College House, 1 Palmerston Crescent South Melbourne, Victoria 3205, Australia Telephone 03 8699 0414 Facsimile 03 8699 0400 Street address Suburb www.racgp.org.au ACN 000 223 807 ABN 34 000 223 807 State Postcode Country Telephone ISSN 0812-9630 The five domains of general practice Communication skills and the patient-doctor relationship © The Royal Australian College of General Applied professional knowledge and skills Population health and the context of general practice Practitioners 2011. All rights reserved. Professional and ethical role Organisational and legal dimensions The opinions expressed in check are not Please allow up to 14 days for delivery of goods. The College must be notified of missing or incorrect orders within 28 days of placing the order. Orders returned necessarily those of the RACGP. Payment details must be received within 28 days of receipt in order to obtain refund or replacement. Prices include GST, postage and handling. Overseas prices exclude GST. Please address all letters concerning Payment type the content to the medical editor. Amex Visa MasterCard Cheque (make payable to the RACGP) Printed by Printgraphics Pty Ltd, 14 Hardner Road, Mount Card number Expiry date Total amount Waverley, Victoria 3149 Telephone 03 9562 9600. / $ Cardholder name (please print clearly) Cardholder signature

FOR OFFICE USE ONLY RAGGP NO. (Bill to) ______BATCH NO. ______ORDER NO. ______INVOICE NO. ______

check 2008–2010 order form ad.indd 1 11/03/11 2:33 PM from the editor check Ear nose and throat

This unit of check looks at a variety of clinical scenarios relating to ear nose and throat (ENT) problems which are commonly seen in general practice. It is important that general practitioners are competent in their assessment of these problems. History and examination should aim to determine the diagnosis, exclude serious differential diagnoses, and identify indications for referral to ENT surgeons and other specialists such as neurologists and allergists, depending on the clinical situation. Management should be evidence based as far as possible. The authors bring a wealth of clinical, research and teaching experience in the area of ENT surgery, otolaryngology, rhinology and neuro- ophthamology. The authors of this unit are: June Choo MBBS, FRACS, otolaryngologist, head and neck surgeon, Royal Victorian Eye and Ear Hospital and The Alfred, Victoria. Her special interests include voice disorders and surgery for snoring and obstructive sleep apnoea Michael Dobson MBBS, FRACS, general otorhinolaryngologist, Royal Victorian Eye and Ear Hospital, and specialist in otology and general ENT, and head and neck surgeon at Box Hill Hospital, Victoria. He has over 25 years experience in otology and paediatric ear disease and he has experience with indigenous ear disease in Alice Springs and South East Asia. His clinical and research interests are middle ear surgery and surgery for deafness Ana Teresa Licup MD, DPBO-HNS, Senior House Medical Officer for ENT, Royal Victorian Eye and Ear Hospital, Victoria. Her special interest is paediatric otolaryngology Mark Paine MBBS, FRACP, Head of Vestibular Investigation Unit, Head of Neuro-ophthalmology Clinic, Royal Victorian Eye and Ear Hospital, Head of Neuroimmunology Unit, St Vincent’s Hospital, Victoria. His special interests include neuro-ophthalmology, neuro-otology and multiple sclerosis Elizabeth Rose MBBS, FRACS, otolaryngologist, Royal Children’s Hospital and Royal Victorian Eye and Ear Hospital, and Senior Lecturer, University of Melbourne. Her area of interest and expertise is paediatric otolaryngology, especially hearing loss Philip Michael MBBS, DLO, FRCSED (ORL-HNS), Rhinology Fellow, Royal Victorian Eye and Ear Hospital Melbourne. His special interests include endoscopic sinus surgery, functional rhinoplasty and medical education Carmel Crock MBBS, FACEM, BLitt, Director, Emergency Department, Royal Victorian Eye and Ear Hospital, Victoria. Her interests are in patient safety and medical error. She is involved in medical education, including undergraduate, emergency medicine and GP education in eye and ENT emergencies. The learning objectives of this unit are to: • display increased confidence in the assessment and management of patients who present with otitis media, otitis externa, vertigo, hoarseness of the voice, sudden hearing loss, allergic rhinitis, chronic sinusitis and epistaxis • recognise the importance of urgent referral in conditions such as stroke presenting as vertigo, certain cases of epistaxis, idiopathic sudden sensorineural hearing loss and malignant otitis externa, and recognise the importance of prompt referral in cases of suspected carcinoma of the , and nasal passages • display increased confidence in performing certain aspects of examination of a patient, such as the head impulse test, Hallpike manoeuvre and Weber test • display increased confidence in interpreting signs on otoscopic examination of common conditions such as otitis media and otitis externa • recognise the importance of certain procedures such as the use of tissue spears in otitis media with perforation, ear canal debridement in otitis externa and nasal packing in epistaxis in the management of these problems in general practice. This issue of check heralds a change in the medical editor. We thank Dr Kath O’Connor as the outgoing medical editor of the check Program for all her work over the years and for her contribution in helping to make this publication successful. I would like to introduce myself as the new medical editor. I look forward to the privilege of working with you in the months to come. I hope this unit will assist you to manage ear nose and throat problems in general practice.

Kind regards

Catherine Dodgshun Medical Editor

2 check Ear nose and throat Case 1

Question 3 Case 1 How would you manage her hoarseness at this stage? Would you ChrisAdriana has has a hoarse a hoarse voi voice ce prescribe medication such as antibiotics or corticosteroids? ChrisAdriana is 58is 50years years of ofage, age he and is a is builder a receptionist who has at comean accounting for his annual firm. Shecardiovascular developed aand hoarse prostate voice 6 healthweeks check.ago and You has find not he been is overweight able to take with any a timebody massoff work index as the(BMI) firm of 29is experiencing kg/m2. At the a closebusy period.of your physicalShe finds exam that asand the testing day progresses, he mentions the that hoarseness he has beenworsens, getting her painstress on levels and off increase in his right and sheknee also and thatexperiences it is interfering a tightness with inwork, her throat.particularly When climbing Adriana uporiginally and down presented ladders 1 andweek carrying after the heavy onset timber of her loads.symptoms, The kneeshe washas treatedbeen swollen for after some with busytwo dayscourses at work.of oral He antibiotics reports playing but does a lot not of feel sport that in her Question 4 hisvoice youth is improving. with previous She wouldknee injuries like to trybut ano ‘stronger’ prior surgery.antibiotic. He is worried about his future as a builder. Could Adriana’s intake of ibuprofen be related to her voice hoarseness? YouAdriana quickly is mindful ascertain of herthat family the knee history is not of currently‘throat swollencancer’, andwhich has her full father range developed of movement in his and 60s you and scheduleis concerned another about appointment her risk. You to note focus from on diagnosisAdriana’s andmedical management record that of shehis kneehas a pain.past history of osteoarthritis of the lumbar spine, for which she says she has been taking ibuprofen on a regular basis.

Question 1 What is the most likely cause for Adriana’s symptoms?

Question 5 What are the main risk factors for ‘throat cancer’? What symptoms or signs would arouse suspicion of cancer?

Question 2

What other diagnoses should be considered? What other features in Question 6 the history would you be interested in? If Adriana had woken up after an operation with hoarseness of the voice, could it be related to the operation? Explain your answer.

3 Case 1 check Ear nose and throat

It is important to take a history relating to: CASE 1 ANSWERS • the onset of hoarseness • whether hoarseness is constant (which tends to occur in Answer 1 structural lesions and traumatic injury) or intermittent (which The most likely cause for Adriana’s symptoms is ‘muscle tension tends to occur in muscle tension dysphonia, dysphonia’, which is a common functional disorder of the larynx. gastro-oesphageal reflux and postnasal drip) Muscle tension dysphonia is a clinical diagnosis with pointers in • aggravating and relieving factors the history as well as findings on laryngoscopy. Laryngoscopy is • associated symptoms such as cough (as can occur in performed both to exclude other pathology and to demonstrate gastro-oesphageal reflux, allergies and as an effect of chronic muscle tension dysphonia. Muscle tension dysphonia usually laryngitis) responds well to voice therapy.1 • risk factors for ‘throat cancer’ (see Answer 4) • determining each of the causes of hoarseness in Answer 2 Feedback above: The majority of hoarseness in the community is due to viral –– voice abuse laryngitis. Hoarseness from viral laryngitis typically lasts up to 3 weeks.2 If hoarseness persists beyond 3 weeks, referral –– symptoms such as heartburn and waterbrash for laryngoscopy should be considered to exclude other –– symptoms such as postnasal drip causes. –– symptoms associated with throat cancer (see Answer 4) –– history of neck trauma Answer 2 –– recent surgery Other diagnoses to consider as a cause for Adriana’s hoarseness include: –– general state of health • structural lesions which can either be benign (eg. nodules, • excluding each of the causes of throat tightness in Answer 2 polyps and papillomas) or malignant (eg. carcinoma of the above. larynx) • gastro-oesophageal reflux Answer 3 Adriana’s hoarseness should be managed by referral to an ear, • postnasal drip nose and throat (ENT) consultant for consideration of laryngoscopy. • irritation from inhaled corticosteroids used to treat asthma Antibiotics are unlikely to be helpful for Adriana’s hoarseness and, • irritation from fumes, dust or cigarette smoke in general, are not helpful in most cases of hoarseness. • autoimmune disease (eg. systemic lupus erythematosus, There is no evidence that empirical use of corticosteroids is sarcoidosis and Wegener granulomatosis) helpful for voice hoarseness.3 Corticosteroids could be helpful in • infection (eg. Candida, or with Staphylococcus aureus) previously diagnosed autoimmune conditions affecting the larynx, • allergy such as systemic lupus erythematosus, sarcoidosis and Wegener granulomatosis. • traumatic injury. Corticosteriods should be considered in airway and The other important symptom which Adriana describes is throat oedema, as can occur in emergency situations, as should transfer tightness. Possible causes for Adriana’s throat tightness include: by ambulance to an appropriate hospital emergency department. • ischaemic heart disease – throat tightness may be atypical pain which could indicate underlying ischaemic heart disease. It is Answer 4 important to exclude ischaemic heart disease by appropriate investigations if this diagnosis is considered It is possible that ibuprofen could exacerbate gastro-oesophageal reflux into the laryngopharynx or contribute to the formation of a • allergy – while allergy is a possible cause for Adriana’s throat haemorrhagic polyp, therefore contributing to voice hoarseness. tightness, in the absence of perioral swelling or rash, allergy is not likely Answer 5 • asthma – asthma can present as throat tightness, though there ‘Throat cancer’ is a nonspecific term and can mean cancer of the are likely to be other symptoms such as cough, shortness of pharynx or larynx. The most common malignancy in the pharynx breath or wheeze present and larynx is squamous cell carcinoma and the main risk factors • globus pharyngis – globus pharyngis is another condition which are smoking and heavy alcohol intake. Other less common risk causes throat tightness but it usually presents with a sensation factors include immunosuppression, exposure to asbestos and of a lump in the throat in the absence of signs. previous radiotherapy.

4 check Ear nose and throat Case 1

Some patients with squamous cell carcinoma of the head and neck region do not have the common risk factors. Symptoms and signs of concern include: • • odynophagia • loss of appetite or weight • otalgia • stridor or a neck mass. If a patient has a persistently hoarse voice and risk factors for malignancy, or symptoms or signs of concern, an urgent referral to an ENT consultant should be made.

Answer 6 Yes, hoarseness of the voice could be related to the operation. Voice hoarseness is not uncommon following surgery where an endotracheal tube or laryngeal mask has been used. The hoarseness is usually due to laryngeal oedema and resolves spontaneously over a period of up to 6 weeks. Hoarseness can also be from a vocal cord granuloma3 or dislocation of the arytenoid cartilage resulting in an immobile vocal fold. It is important that the patient be referred to an ENT specialist for investigation with flexible laryngoscopy. Voice hoarseness after surgery can also be due to trauma to the recurrent laryngeal nerve. Some surgical procedures are more likely to be associated with trauma to the recurrent laryngeal nerve. These include: • cervical laminectomy via the anterior approach • thyroid surgery • carotid endarterectomy • cardiac surgery • surgery for oesophageal cancer. Recurrent laryngeal nerve palsy could resolve and while awaiting recovery, voice therapy could improve function. Treatment of vocal cord granuloma usually includes a combination of voice therapy, acid suppression and surgical excision.

5 Case 2 check Ear nose and throat

Further Information Case 2 Jean’s Weber test lateralises to the left ear. Your otoscopic Jean’s sudden loss of hearing examination reveals a nonobstructed ear canal and a normal tympanic membrane. Jean, a busy housewife of 41 years of age, presents to you in the afternoon after having woken up with a Question 4 blocked feeling in her right ear. She had tried to ring her husband at work and thought that the phone What is the likely diagnosis? What is your next step in management? was dead. Before ringing the telephone company she got her son to check the phone and her son said that the phone was working fine. Jean then put the handpiece to her left ear and she was able to hear with her left ear. She tells you that 1 week ago she had a minor upper respiratory sniffle which resolved without any treatment.

Question 1 What would be your first step on examination?

Question 5 Question 2 If Jean’s Weber test had lateralised to the right ear, what would this Describe how you would perform this office test. indicate?

Question 3 What should you do if you do not have a tuning fork?

6 check Ear nose and throat Case 2

• There is consensus that immediate steroid treatment increases the CASE 2 ANSWERS chance of recovery. Prednisolone is used at a starting dose of 1 mg/kg and continued for 10 days; the dose is tapered after 3 days of maximum dose4 Answer 1 The first step on examination is to perform a Weber tuning fork test. • Antiviral medications have been advocated but there is no hard This simple office test will quickly differentiate between conductive evidence that there is any significant benefit deafness and sensorineural deafness. An otoscopic examination • Between 20–30% of cases will improve spontaneously. should also be performed. Figure 1 shows a normal tympanic membrane.

Answer 2 The Weber test is performed using a tuning fork of 512 Hertz (middle C). Strike the tuning fork and then place the stem firmly on the midpoint of the patient’s forehead. Ask the patient which ear the sound of the fork is louder in. The test is interpreted as follows: • normal hearing – the sound is heard in the midline • conductive deafness – the sound is heard in the deaf ear • sensorineural deafness – the sound is heard in the nondeaf ear.

Answer 3 Ask the patient to hum. Ask in which ear they hear the hum. The same interpretation principles of the Weber test apply.

Answer 4 If the Weber test lateralises to the nondeaf ear, this suggests sensorineural hearing loss. In Jean’s case, as the hearing loss has been sudden, the most likely diagnosis is idiopathic sudden sensorineural hearing loss (ISSHL). The next step would be to obtain Figure 1. A normal tympanic membrane. Reproduced with permission from immediate advice from an ENT surgeon either by phone or by a hawkelibrary.com referral that day. Further information on ISSHL is presented below. • Incidence – this varies between 5–20/100 000 per annum • Aetiology – there is ongoing debate about the cause. A viral theory is plausible, however, a microvascular event involving the cochlear-vestibular circulation may explain a proportion of presentations • About 40% of cases report associated balance disturbance • Approximately 10% of acoustic neuromata present with sudden sensorineural deafness therefore part of the diagnostic workup will involve a magnetic resonance imaging (MRI) scan • ISSHL is an emergency needing immediate treatment

Figure 2. Ear obstructed with wax. Reproduced with permission from hawkelibrary.com

7 Case 2 check Ear nose and throat

Figure 3. Otitis media with effusion. Reproduced with permission Figure 4. Early herpes zoster vesicle on the ear drum. Author’s image from hawkelibrary.com

Answer 5 If the Weber test lateralises to the right, this indicates that there is a conductive deafness which can be caused by obstruction of the external ear canal (Figure 2) or by pathology in the middle ear (Figure 3). If pathology in the middle ear was the cause, Jean’s tympanic membrane would look abnormal due to the presence of middle ear fluid reflecting eustachian tube blockage secondary to upper respiratory inflammation. If this is the case, treatment should be with nasal decongestants, either topical or systemic. If otalgia was present, this might represent otitis media.

Feedback Another possible cause of sensorineural deafness is shown in Figure 4. This is an early herpes zoster vesicle on the ear drum. The patient presented with the onset of severe otalgia and a sensorineural deafness before proceeding to develop a facial nerve palsy (Ramsay Hunt syndrome). In this instance antiviral medication and steroids together are the first line treatments.

8 check Ear nose and throat Case 3

Case 3 Robbie’s ongoing ear problems Robbie, 5 years of age, is brought to see you with a discharge coming from his left ear (otorrhoea). He has a cold this week and cried with earache during the night, but went back to sleep after taking paracetamol. You check his medical record and note that he has had about 2–3 episodes of acute otitis media (AOM) every winter for the last 2 years, and has been treated with antibiotics for each of these episodes. He started school this year and has had several colds as well as gastroenteritis since then. It is May and this is his third ear infection since February. His mother is alarmed as he has never had otorrhoea before. Robbie is alert and interactive with a temperature of 37.8°C. His cervical nodes are slightly enlarged, throat examination is normal and chest is clear. Otoscopic findings are shown in Figure 5. Robbie’s right ear. Reproduced with permission from hawkelibrary.com Figure 5 and 6.

Question 1 What is your diagnosis?

Question 2 What is meant by the term recurrent acute otitis media (AOM)? Figure 6. Robbie’s left ear. Reproduced with permission from hawkelibrary.com

9 Case 3 check Ear nose and throat

Question 3 Question 6 What are the organisms that commonly cause AOM? How will you treat Robbie’s current episode?

Further information Robbie has a new sister, Alice, age 4 months. Their mother Question 4 has been on maternity leave, but plans to return to work next What else would you like to know before deciding on a treatment month. She would like to know whether it is likely that Alice plan? will have a lot of problems with her ears as well, and if there is anything she can do to prevent this.

Question 7 What will you tell Robbie and Alice’s mother?

Question 5 How would you clean his left ear?

Question 8 When would you ask Robbie to come back for review?

10 check Ear nose and throat Case 3

Further information Question 9 You see Robbie 2 weeks later. His pain and discharge settled Describe the otoscopic findings and what your advice is now. within 48 hours of treatment, and his mother is not concerned about his hearing. Otoscopic findings are shown in Figure 7 and 8.

Further information Robbie returns 5 weeks later. The night before Robbie had a bad night and is still in pain. His mother has been for his parent-teacher interview, and the teacher has concerns that Robbie is not paying attention, and he is not learning his spelling as well as he was earlier in the year. His mother reports that he often seems grumpy and does not do as he is told, especially if he is watching television. He has a constantly runny nose and wipes it but cannot blow it efficiently. He often comes into his parents’ bed at night, which he has not done since he was a toddler. They note that he snores and mouth Figure 7. Robbie’s right ear. Reproduced with permission from hawkelibrary.com breathes but does not appear to stop breathing. Otoscopic findings are shown in Figure 9 and 10.

Figure 8. Robbie’s left ear. Reproduced with permission from hawkelibrary.com Figure 9. Robbie’s right ear. Reproduced with permission from hawkelibrary.com

11 Case 3 check Ear nose and throat

Question 12 What are the indications for insertion of middle ear ventilation tubes?

Question 13 Figure 10. Robbie’s left ear. Reproduced with permission from hawkelibrary.com What is the efficacy of prophylactic antibiotics for the prevention of otitis media in young children?

Question 10 What is your diagnosis for each ear?

Question 14 What is the efficacy of adenoidectomy for a child with middle ear disease? Question 11 What is your management for Robbie now?

12 check Ear nose and throat Case 3

Answer 5 CASE 3 ANSWERS This can be done with suction if available, or with mopping with tissue spears. His mother can continue with use of tissue spears at Answer 1 home as well (Figure 11). The diagnosis pertaining to the right ear is AOM. The tympanic membrane is red and injected, and there is loss of the prominence of the anterior process of the malleus, consistent with pus under pressure. The diagnosis pertaining to the left ear is likely to be AOM with perforation. The ear needs to be cleaned to assess the tympanic membrane (the process by which this is performed is described in Answer 5). The diagnosis of AOM may be difficult to make, especially in children younger than 3 years who have small external auditory canals, may not be cooperative and/or have wax obscuring the view. In a comparison of doctors it was found that the accuracy in diagnosing whether or not there is middle ear fluid was: ENT surgeons – 74%; paediatricians – 51%; GPs – 46%.5 Using pneumatic otoscopy or a tympanogram can improve the diagnostic accuracy.

Answer 2 Recurrent AOM is defined as three or more episodes in 6 months, or four or more in 12 months.6 This is an indication that a child is having more infections than would usually be expected.

Figure 11. Tissue spears. Author’s image Answer 3 The organisms that commonly cause AOM are Streptococcus pneumoniae, Haemophilus influenzae – nontypeable, and Moraxella Answer 6 catarrhalis.5 Treatment of AOM involves the following.

Answer 4 Adequate analgesia Further questions to ask include the following. Usually with an oral agent such as paracetamol. Topical anaesthetic • Is this Robbie’s first year of having a lot of contact with other drops such as amethocaine, benzocaine or lidocaine have some children? (If he has not had previous exposure to infections efficacy at 30 minutes after administration, but not more than through childcare then he is likely to continue having colds and paracetamol once they have been absorbed.8 Topical anaesthetic ear infections) drops should not be administered when there is a perforation as they • Does Robbie have a past history of other infections, such as may enter the inner ear and cause vertigo. pneumonia, suggestive of an immune deficiency? Antibiotic therapy • Does he swim regularly in a swimming pool? (Although there is The current recommendations are that children older than 2 years evidence that for Aboriginal children in remote communities in who are not very ill (ie. no systemic features such as significant fever Western Australia, swimming in pools can reduce ear infections, or vomiting) can be treated with observation only; the parents can this mainly relates to children with perforated tympanic be given a prescription to be filled if the child is still in pain after 48 membranes and chronic otorrhoea. Although swimming on the hours.9 However, there are groups of children who should be treated. surface of the water is not usually associated with eustachian These include: tube and middle ear problems, swimming more than 60 cm below • children 2 years of age or younger, because: the surface is sufficient to insufflate infected nasopharyngeal secretions into the middle ear during an upper respiratory –– they are not able to describe their symptoms infection.7 If Robbie is having regular swimming lessons his –– they are more likely to have suppurative complications such parents could try keeping him out of the pool for winter) as meningitis and mastoiditis • Are there concerns about his hearing either at home or in school? –– they are less likely to improve spontaneously

13 Case 3 check Ear nose and throat

• children with severe illness with pain, or a tympanic membrane • vaccination with the polyvalent pneumococcal vaccine reduces the perforation (these imply a more virulent organism) incidence of AOM by 8%. • a child with known immunodeficiency • indigenous children, including Aboriginal, Torres Strait Islander and Answer 8 Maori and other Pacific Islander children Robbie should be reviewed in 2 days if he is no better. At that stage, if the AOM has not resolved, institute change to an antibiotic such as • children with a cochlear implant. amoxycillin/clavulanate. The usual antibiotic recommended is amoxycillin, 15 mg/kg up to Robbie should also be reviewed at the 2 week mark to ensure that 500 mg three times per day for 5 days. If the child has an allergy the perforation of the tympanic membrane has healed. Note that at to penicillin (excluding immediate hypersensitivity), the alternative 2 weeks following an episode of AOM, 70% of children will still have medication is cefuroxime in a dose of 10 mg/kg up to 500 mg twice a middle ear effusion but most perforations will have healed. per day for 5 days, or ceflacor in a dose of 10 mg/kg up to 250 mg three times per day for 5 days.10 The efficacy of ceflacor is thought to be similar to that of amoxycillin but it may be less as it penetrates Answer 9 middle ear mucosa less efficiently.6 The infection in his right ear is resolving, and there is some air in the With a perforation of the eardrum, topical antibiotic drops are useful middle ear, indicating that the eustachian tube is starting to open and both to treat the middle ear and also to treat secondary otitis externa function again. if present. The recommended antibiotics are quinolones such as In his left ear, there is still a perforation of the tympanic ciprofloxacin with or without steroid, which are not ototoxic when there membrane, but it is small and may close. There is also a patch is a tympanic membrane perforation.6 If the tympanic membrane is of tympanosclerosis in the tympanic membrane, consistent with intact there is no advantage in treating with topical antibiotics. previous infection. As Robbie has a perforation this ear should be kept dry. As there is Your advice to Robbie’s mother should be to keep the left ear dry and infection and fluid in both his ears Robbie may not be hearing well. return in about 3 months to check the perforation and the fluid. His mother should be warned of this and asked to notify the teachers at school to report concerns about his hearing or behavioural Answer 10 changes which could suggest hearing difficulties. In his right ear, there is a middle ear effusion. It is opaque and the anterior process of the malleus is prominent. The tympanic Answer 7 membrane is retracted, especially posterosuperiorly. Alice’s risk of recurrent otitis media depends on the presence of In his left ear, the perforation has healed and there is AOM; the recognised risk factors for otitis media. These are:6 tympanic membrane may perforate again. • age – especially 6–11 months • race – eg. Indigenous Australian children. There are differences Answer 11 in the eustachian tube and immunological response, but Robbie has had a further ear infection and has an ongoing middle socioeconomic factors are also important ear effusion. These factors are likely to be affecting his hearing and • craniofacial abnormalities – including cleft palate and those school work. caused by Down syndrome Management could involve: • genetic – both anatomical and immunological • use of amoxycillin/clavulanate in case Robbie had a resistant • birth order – children with older siblings are more likely to have Pneumococcus as he has had a recent infection treated with infections than the first born. amoxycillin There are recognised environmental factors, and Alice’s mother can • referral for audiology help control these by implementing the following: • treating his nose – saline sprays help to clear the mucus, and • reduce contact with people with upper respiratory infections, steroid sprays help to reduce the size of the adenoids, but there is especially large group childcare centres no evidence that either will improve the eustachian tube function and clear fluid from the middle ear.11 • avoid tobacco smoke both during and after pregnancy. Children exposed to passive smoking are more likely to have recurrent otitis media and have middle ear effusions that persist for longer Answer 12 The guidelines published in the American Academy of Family • breastfeed for at least 6 months, preferably 12 months. If bottle Physicians, American Academy of Otolaryngology-Head and Neck fed, prop the baby up as milk can reflux into the ear if lying flat, Surgery and American Academy of Pediatrics Subcommittee on Otitis causing inflammation Media With Effusion propose that indications for insertion of middle • avoid pacifiers/dummies – this possibly increases the risk of AOM ear ventilation tubes include:12 by inadvertent sharing in childcare centres

14 check Ear nose and throat Case 3

• chronic otitis media with effusion for 4 months or longer with conductive hearing loss. The level of hearing that is significant is not known, and probably differs for each child • significant hearing loss • known language delay • learning/intellectual problems • visual impairment in addition to hearing loss from chronic otitis media with effusion • damage to the tympanic membrane with retraction pockets to prevent permanent ossicular erosion. Middle ear ventilation tubes are also effective in preventing recurrent acute otitis media, and for the treatment of complications of acute otitis media such as facial nerve paresis and meningitis.6

Answer 13 Treatment with prophylactic antibiotics (usually a single daily dose of amoxycillin 20 mg/kg) has some efficacy, however, due to concerns about antibiotic resistance, prophylactic antibiotics are reserved for children under special circumstances.13 This includes children who are a poor risk for general anaesthesia, or those who have had tubes inserted and have persistent infections.

Answer 14 It is usually recommended that adenoidectomy be performed if the child needs a second set of ventilation tubes, unless there is significant nasal obstruction, in which case it may be performed earlier. Adenoidectomy has not been shown to be effective in preventing recurrent AOM.6

15 Case 4 check Ear nose and throat

Question 3 Case 4 What ‘red flags’ suggest stroke syndrome? Nigel is experiencing vertigo and nausea Nigel is 52 years of age with a background history of hypertension, hypercholesterolaemia, impaired glucose tolerance and migraine, and a long history of heavy cigarette use. He presents to your clinic after the onset of severe vertigo and vomiting. The vertigo came on over a 20 minute period 5 hours earlier in the day. There is no history of previous vertiginous episodes even with his previous migraines. On examination, you note that Nigel is pale and sweaty with normal vital signs. You note that he has a left beating nystagmus which is enhanced on left gaze. The head impulse test is abnormal to the right with an abnormal corrective refixation movement. There were no other neurological signs evident. Hearing is clinically normal and otoscopy is clear. Question 4 You perform a head impulse test on Nigel to add weight to your Question 1 suspected diagnosis. Describe how to perform a head impulse test. What is the most likely clinical diagnosis on the basis of the information presented so far?

Question 5 Question 2 What further examination could you perform to help identify the What is the differential diagnosis? cause of Nigel’s vertigo?

16 check Ear nose and throat Case 4

Further information Question 9 The Romberg test is markedly positive as Nigel would have What treatment could you give in the postacute setting? fallen if he were not held. Nigel is having considerable difficulty walking and is tending to veer to the right. The Hallpike manoeuvre induced left beating horizontal nystagmus, which is more accentuated than the spontaneous nystagmus described above. There is no latency to the nystagmus and it persisted throughout the positioning.

Question 6 What is the significance of the examination findings described above?

Further information After significant initial improvement over several weeks, Nigel experiences further recurrent episodic vertigo.

Question 10 Question 7 What are the possible causes of recurrence? What treatment would you give this patient in the acute setting?

Question 8 Should you request imaging in Nigel’s case? What features would suggest that the patient ought to be considered for imaging study or admitted to hospital?

17 Case 4 check Ear nose and throat

Table 1. Meniere disease CASE 4 ANSWERS The American Academy of Otolaryngology-Head and Neck Surgery Criteria for Diagnosis of Meniere Disease16 Answer 1 • Recurrent spontaneous and episodic vertigo – a definitive spell of The most likely clinical diagnosis is acute vestibular syndrome. vertigo lasting at least 20 minutes, often prostrating, accompanied Acute vestibular syndrome is a description of a clinical syndrome by disequilibrium than can last several days; usually nausea or vomiting, or both; no loss of consciousness. Horizontal rotatory which usually implies an acute peripheral vestibulopathy, however, nystagmus is always present occasionally a central disturbance can mimic this syndrome. The • Hearing loss (not necessarily fluctuating) presumptive cause of acute vestibular syndrome is vestibular neuritis, otherwise known as vestibular neuronitis or viral neurolabyrinthitis. This • Either tinnitus or aural fullness, or both clinical syndrome is commonly thought to be related to reactivation of Certain Meniere disease Definite disease with histopathological herpes simplex virus infection of vestibular nerve ganglion (also known confirmation as Scarpa ganglion). Definite Meniere disease Two or more definitive episodes of vertigo with hearing loss, plus tinnitus, aural fullness or both Answer 2 Probable Meniere disease Only one definitive episode of vertigo and The differential diagnoses are outlined below. the other symptoms and signs Stroke Possible Meniere disease Definitive vertigo with no hearing loss, or hearing loss with nondefinitive The vestibular pseudoneuritis syndrome can appear very similar disequilibrium to vestibular neuritis. Vestibular pseudoneuritis is due to brainstem/cerebellar ischaemia (ie. ischaemia in the posterior inferior cerebellar artery or anterior inferior cerebellar artery territories). Table 2. Vestibular migraine Stroke should be considered when there are vascular risk factors, the Neuhauser criteria for diagnosing migrainous vertigo17 presence of any one of the red flags or abnormal signs on examination of the cranial nerves. A careful ocular motor examination can help Definite – all of the Probable – all of the following distinguish central vestibular pseudoneuritis and vestibular neuritis.14 following features features The clinical signs in this case synopsis do not suggest a stroke Episodic vestibular Episodic vestibular symptoms of moderate syndrome. There are a number of red flags to consider in patients symptoms of moderate severity, such as: presenting with vertigo and nausea that are not present in Nigel’s severity, such as: • vertigo (spinning) case – these are outlined in Answer 3. • vertigo (spinning) • positional vertigo • positional vertigo • head motion intolerance Meniere disease • head motion intolerance While Meniere disease, which is frequently overdiagnosed, can cause Migraine according to One or more of the following features: acute vertigo, it is not the likely cause in this case because: International Headache • migrainous headache • usually the vertigo of acute Meniere disease attacks would not Society criteria* • migrainous symptoms during vertigo last as long as 5 hours, though the after effects of nausea, • migraine specific triggers of vertigo disequilibrium and ear symptoms may last hours to days (eg. specific foods, lack of sleep) • recurrent episodes are required to make the diagnosis • response to antimigraine therapy • associated ear symptoms (eg. fluctuating low frequency sensorineural hearing loss, aural fullness and tinnitus) while not One or more of the Other diagnoses excluded by appropriate following features during tests invariably, are often present15 – outlines the diagnostic Table 1 at least two vertiginous criteria for Meniere disease. attacks: • migrainous headache Benign paroxysmal positional vertigo Episodes of vertigo in benign paroxysmal positional vertigo (BPPV) are • photophobia usually: • phonophobia • provoked by a change in position of the head or change in • migraine aura posture Other diagnoses excluded by appropriate tests • brief and not accompanied by vomiting and not associated with spontaneous nystagmus before provocation. * Headache Classification Committee of the International Headache Society. Classification and diagnostic criteria for headache disorders, cranial neuralgias and facial pain. Cephalalgia 1998;8(Suppl 7):1–96.

18 check Ear nose and throat Case 4

Migraine to the opposite side. The test may then be repeated to assess for Migraine can cause acute vertigo, however, usually there are no clinical fatigability of the response (see Resources). signs and there is usually, but not invariably, significant headache A positive Hallpike test has the following characteristics and suggests a present. Consider a diagnosis of migraine where there is a past or diagnosis of BPPV: family history of migraine. Like Meniere disease, recurrent episodes • brief latency – in BPPV, there is usually a brief latency of several are required before a diagnosis of vestibular migraine can be made. seconds before the onset of nystagmus and it usually lasts 10–20 The criteria for diagnosing migrainous vertigo are listed in Table 2. seconds • nystagmus – usually torsional (rotational around the anteroposterior Answer 3 axis of the eye globe) but may be horizontal. In BPPV, on performing Any one of the following red flags suggests the possibility of a stroke or the Hallpike manoeuvre, there is usually upbeating, torsional other central lesion: nystagmus arising from otolithic debris in the ipsilateral posterior • acute unaccustomed headache semicircular canal. Horizontal nystagmus on positioning suggests • inability to stand or walk that the lateral canal is affected. Downbeating, torsional nystagmus which occurs on positioning indicates that the ipsilateral anterior • the presence of spontaneous direction changing nystagmus semicircular canal is affected • spontaneous vertical nystagmus • reversal – upon sitting after a positive manoeuvre, the direction of • a normal head impulse nystagmus is reversed for a brief period of time • the presence of additional focal neurological signs. • fatigability – repetition of the test will result in less nystagmus each time. Answer 4 The head impulse test is performed by rapidly rotating the patient’s Answer 6 head while the patient is instructed to fixate on a central object such as The presence of a positive Romberg test (a Romberg sign) indicates the examiner’s nose. When the vestibulo-ocular reflex is intact, an equal loss of proprioceptive or vestibular input, which, in the context of Nigel’s and opposite eye movement keeps the eyes stationary in space. An presentation, is supportive of a diagnosis of vestibular neuritis. On the abnormal result is indicated by failure to maintain fixation. In this case, other hand, when a central lesion is present where the cerebellum movement of the eyes occurs then the patient will suddenly perform is affected, such as in the case of a stroke, while the patient is also corrective refixation back to focus on the central object. An abnormal unstable at rest, they are not made more unstable when performing a test suggests an acute vestibular syndrome due to vestibular neuritis Romberg test (this constitutes a negative Romberg test). and a normal test usually suggests the symptoms are due to a central In cases of acute vestibular syndrome such as Nigel’s, there is lesion such as stroke, though this is not always the case and must be spontaneous horizontal nystagmus before performing the Hallpike interpreted in context (see Resources). manoeuvre, which suggests a peripheral cause of vertigo other than BPPV. In acute vestibular syndrome caused by vestibular neuritis, Answer 5 nystagmus is often noted while performing the Hallpike manoeuvre Further examination could consist of the following: as well as it being present before performing the maneouvre. In these • Romberg test – to perform this test, the patient stands erect with cases the Hallpike manoeuvre will not result in latency, reversal and their feet close together and eyes closed and is observed for fatigability in vestibular neuritis as it tends to in cases of BPPV. 1 minute. Romberg sign is said to be positive if the patient becomes In cerebrovascular causes of vertigo, spontaneous nystagmus can unsteady in this position occur and it will usually be vertical or direction changing. On performing • bedside assessment of hearing the Hallpike manoeuvre, there is upbeating, downbeating or direction • assessment of gait and preponderance to veer to one or other side changing nystagmus but no latency, reversibility or fatigability. • Hallpike manoeuvre – take care with the manoeuvre in patients who have neck disease and avoid performing the manoeuvre in patients Answer 7 with major cervical spine instability. After explaining to the patient Once a diagnosis of acute vestibular neuritis is made, treatment could what you are going to do, the patient sits upright with the head consist of the following: rotated 30–45 degrees laterally. The patient is then rapidly moved • symptomatic treatment with antiemetics and fluids for rehydration into a supine position on the examination couch with the head if necessary hanging over the end of the couch or a pillow placed behind the • benzodiazepines may be useful in reducing the intensity of vertigo shoulders. The patient’s head is supported either by the examiner for the first 24–48 hours but should be used sparingly after this or by the couch if a pillow is placed behind the shoulders. The time period examiner then observes the patient for nystagmus and asks about • steroids – a course of steroids similar to that given for Bell palsy vertigo. Then the test is repeated with the patient’s head turned should be considered18,19

19 Case 4 check Ear nose and throat

• antiviral therapy – there is controversy over the use of antiviral Decompensation therapy in this situation, although the existing evidence does not After initial improvement from vestibular neuritis, decompensation support its routine use.19 can occur and the reasons are often not clear. Formal vestibular investigations and vestibular physiotherapy may be useful in Answer 8 clarifying this clinical scenario. In Nigel’s case, computed tomography (CT) or MRI brain would If the original bout of acute vertigo lasts less than 24 hours, and not be considered absolutely necessary. Nigel could be referred the Hallpike manoeuvre is repeatedly negative, then other possible to hospital for imaging to look for evidence of stroke if there is in diagnoses such as vestibular migraine, Meniere disease or even any doubt about the diagnosis, or for rehydration if his vomiting is ‘recurrent vestibulopathy’ (a term used for recurrent unexplained severe. vertigo) may explain recurrent symptoms. If there is significant suspicion of a stroke, then the patient ought Migraine to be admitted for observation and investigation. The presence of With migraine, vertigo may occur with or without headache. any one of the red flags described in Answer 3 with the presence of vascular risk factors suggest a possible stroke, in which case Meniere disease a neuroimaging study, preferably MRI, needs to be ordered. Additional clinical features such as those outlined in Table 1 and 2 Frequently a CT brain is ordered in this situation, however, a CT should sort out whether migraine or Meniere disease is most likely. scan is not sufficient to rule out a stroke in the posterior fossa. A CT brain scan may reveal a large cerebellar hemisphere infarct or Recurrent vestibulopathy a posterior fossa haemorrhage. Usually the CT brain in the clinical A term used for recurrent unexplained vertigo. scenario described above is normal. Certainly if there is no, or limited, access to MRI, then it would be reasonable to get a CT Recurrent vestibular neuritis brain and consider transfer to another institution with MRI access. If Recurrent vestibular neuritis may occur but is uncommon. there is doubt about the diagnosis, admission for observation would be wise. Vascular vertigo In Nigel’s case, vascular vertigo (vertigo due to cerebrovascular Answer 9 disease) might be suspected with the occurrence of recurrent Following a bout of vestibular neuritis, recovery usually begins after vertigo, particularly in the presence of vascular risk factors. Usually a few days, although the patient may be rendered bedbound or the recurrent symptoms would be brief, lasting seconds to minutes, housebound for up to 1 week. and would not ordinarily continue beyond 6–8 weeks without other transient ischaemic attack manifestations or permanent posterior As the patient recovers from the acute disturbance, vestibular circulation territory infarction. adaptation exercises or even formal vestibular physiotherapy should be considered to aid further recovery over the ensuing weeks or Perilymph fistula or autoimmune inner ear disease months. Perilymph fistula and autoimmune inner ear disease would be other much less common causes of recurrent vertigo and would usually Answer 10 be associated with hearing loss. Possible causes of recurrent vertigo include the following.

Postvestibular neuritis benign positional vertigo The recurrence of vertigo after a few weeks in Nigel most likely indicates postvestibular neuritis benign positional vertigo. Vestibular neuritis frequently involves the superior division of the vestibular nerve resulting in damage/dysfunction to the afferents from the superior and lateral semicircular canals and sparing the posterior semicircular canal afferents. Therefore any otolithic debris dislodged by the acute vestibular insult may accumulate in the posterior semicircular canal and give rise to benign positional vertigo. This diagnosis may be confirmed by performing a Hallpike manoeuvre, which will be positive. It is treated satisfactorily with a particle repositioning manoeuvre such as the Epley (see Resources) or Semont manoeuvre.19,20

20 check Ear nose and throat Case 5

Question 3 Case 5 What features would you be interested in on examination? John’s nosebleed John is 65 years of age and presents to your clinic with a sudden onset of a right sided nosebleed. He has had no significant nasal or sinus problems in the past. He has a past history of mild hypertension and coronary heart disease and takes aspirin 100 mg/day, atorvastatin 10 mg/day and ramipril 10 mg/day.

Question 1 What are the predisposing factors for epistaxis? Which of these are you concerned about in John?

Question 4 How would you differentiate an anterior and posterior nasal bleed?

Question 2 What other features would you be interested in when taking further history from John? Question 5 What are the steps in treating epistaxis?

21 Case 5 check Ear nose and throat

Further history You advise John to pinch the soft part of his nose with pressure CASE 5 ANSWERS exerted toward his face with his head tilted slighty forward for a period of 5 minutes but John continues to bleed through his Answer 1 mouth. On examination, there is blood oozing from both nostrils There are various local and systemic conditions which can predispose but more profuse on the right side. There is difficulty visualising to epistaxis. These include: the nasal cavity because of continuous bleeding and you decide • local conditions – conditions such as trauma, structural to pack the nose blindly with nasal tampons. Despite this abnormalities (eg. septal deviation/perforation); inflammatory intervention, John continues to bleed into the oropharynx. diseases (eg. infection, allergy, granulomatous disorders); tumours and vascular malformations21 Question 6 • environmental conditions – the decrease in ambient humidity What packing materials can you use to stop John’s bleeding? during winter causes a drying effect of the nasal mucosa that impairs wound healing and enhances crust formation.22 This encourages local digital trauma, and a vicious cycle of trauma and inflammation ensues • medications – such as intranasal steroids used for allergic rhinitis, aspirin, anti-inflammatories, clopidogrel and warfarin, and drugs such as cocaine used on a long term basis, may predispose to epistaxis • systemic factors – such as deficiencies in coagulation, vascular diseases and cardiac/pulmonary diseases (the latter, as a result of the combination of increased intravascular pressure and medically-induced coagulopathy) have been associated with more severe epistaxis (as well as higher risk for posterior nasal bleeds).21 Investigation for coagulation disorders should be considered in recurrent and multifocal bleeds. Question 7 John has one important predisposing factor for epistaxis, which is his use of aspirin. While John has hypertension, the association between What situations would prompt you to send a patient with epistaxis to hypertension and severe epistaxis is still debateable.21 a hospital emergency department?

Answer 2 It is important to ask John about the initial laterality, duration and amount of blood loss and whether he has had nose bleeds before. It is also important to ask him questions directed at establishing the cause of his nosebleed, such as a recent common cold, nasal blockage, discharge or crusts, local digital trauma, use of medications, intake of alcohol and drugs, past history of allergic rhinitis and systemic disease and whether he has had any bleeding elsewhere. It is also important to ask him questions directed at identifying the source of bleeding, namely whether it is from an anterior nasal source or posterior nasal source. To elicit this, ask where the bulk of the bleeding seems to be presenting (into the throat or out through the nose), as well as about triggers and response to any first aid applied.

Answer 3 Important features on examination include assessment of haemodynamic status and examination of John’s nose to identify the source of the bleeding. The rest of the examination depends on features elicited in the history. In cases of significant bleeding, history, examination and treatment will proceed concurrently.

22 check Ear nose and throat Case 5

Answer 4 Recurrent epistaxis associated with persistent nasal obstruction There are some factors in the history that will guide you in identifying should be further evaluated for neoplastic causes. Clinical signs the source of the bleeding as being either anterior or posterior. These of skull base tumours (eg. cranial nerve palsies), intracranial are outlined in Table 3. spread and neck node metastases can suggest malignancy and The source of the bleeding can be differentiated by two means on should be referred immediately for imaging and consult with an examination. otolaryngologist or neurosurgeon. A young, male patient with reoccurring significant nosebleeds should be investigated for • Response to first aid – this includes pinching the nostrils together, juvenile angiofibroma. or response to a properly applied anterior nasal pack. More than 90% of nasal bleeding arises from the ‘Little area’ in the anterior Answer 5 nasal septum.23 Anterior epistaxis usually responds to pressure from pinching the nostrils together or a properly applied anterior Most cases of epistaxis are self limiting and patients rarely seek nasal pack. If the bleeding arises from another source, such as the medical help. Patients with spontaneously resolving epistaxis are branches of the sphenopalatine artery posteriorly, these manoeuvres advised to avoid local trauma and avoid nose blowing, hot drinks will be ineffective. If a patient continues to bleed, particularly with and alcohol for 12 hours and are instructed on first aid for epistaxis the blood running down the pharynx despite a properly applied should it recur. Measures to improve nasal hydration such as anterior pack, and coagulation defects have been ruled out or topical antibiotic/antiseptic ointments and air humidifiers may be 21,22 corrected, a posterior bleed must be suspected helpful. • Inspection of the nasal cavity – this requires a good light source, a Below is a stepwise treatment of epistaxis in the office setting: nasal speculum and suction equipment if available, and even this • first aid in mild cases where an anterior epistaxis is suspected allows examination of only the anterior nasal cavity. Locating the – this involves pinching the nostrils together, between the source of bleeding is often difficult, especially when confronted patient’s thumb and index finger, with pressure applied toward with a profusely bleeding patient the face. This is performed in a position with the patient’s head The source of the bleeding can also be indentified with . slightly forward for a total of 5 minutes continuously. In addition, In a well anaesthetised and decongested nose endoscopic evaluation someone else can apply an ice pack to the bridge of the nose will often allow definite source identification.24,25 • patients with persistent bleeding should be assessed for the need for resuscitation and the usual ‘ABCs’ must be performed Feedback when necessary. The attending health providers should always In patients with recurrent, profuse nosebleeds who have protect themselves by using the ‘three Gs’ – gowns, goggles mucocutaneous telangiectasias, a possible diagnosis of and gloves. Epistaxis can potentially deteriorate from a mild hereditary haemorrhagic telangiectasia (HHT – Osler disease) bleed to an uncontrollable exsanguination should be considered. It has an incidence of 2–4 /100 000 and • nasal toilet and evacuation of clots if a suction device with is an inherited disorder through an autosomally dominant gene. appropriate suction tips is available The disease is characterised by mucocutaneous telangiectasia • decongestion and anaesthesia of nasal mucosa with and arteriovenous malformations brought about by the cophenylcaine (lignocaine hydrochloride plus phenylephrine) absence of smooth muscle and elastic tissue in the vascular helps to control low flow minor bleeds, and permits a thorough endothelium. Epistaxis is the most common presentation of inspection of the nasal cavity to determine the specific site of HHT.21,26 the bleeding vessel. Cophenylcaine soaked in ribbon gauze is introduced into the nasal cavity. Pay attention to recommended Table 3. Factors in history to guide you in identifying doses for lignocaine, especially when dealing with children and source of bleeding24 older patients Factors Anterior epistaxis Posterior epistaxis • for anterior epistaxis – you can perform chemical cauterisation Bleeding presentation Anterior nosebleed but can Mainly with silver nitrate sticks or electrocautery. Chemical have oropharyngeal bleed oropharyngeal cauterisation is safely performed after the nose has been in severe cases bleed toileted, decongested and anaesthetised. Do not cauterise Triggering factors Bleeding following anterior No known blindly, and avoid cauterising both sides of the septum nose trauma triggering factors simultaneously (Figure 12) History of bleeds Previous epistaxis with Previous posterior • nasal packing – anterior with or without posterior packing (this history of anterior septal nasal bleed is described further in Answer 6) cauterisation • in the case of refractory epistaxis refer the patient to a hospital Response to anterior Yes No emergency department or ENT specialist. nasal pressure

23 Case 5 check Ear nose and throat

IT IT Septum Septum

A B C

Figure 12. A) Arrow pointing to source of right anterior epistaxis after the nose has been decongested; B) Right anterior epistaxis after being cauterised with silver nitrate; C) Cauterised area covered with oxidised regenerated cellulose (Surgicel®). IT = inferior turbinate. Author’s image

Answer 6 Packing materials include absorbable or nonabsorbable types. Absorbable materials include oxidised regenerated cellulose (Surgicel®) and gelatin foams (Gelfoam®). The mechanism of action is not well described but oxidised regenerated cellulose is believed to form a scaffold for platelets to attach to. The effect of gelatin foams on the other hand is thought to be physical rather than an alteration of clotting mechanism. Nonabsorbable packs include polyvinyl acetyl sponge (Merocel®), impregnated ribbon gauze (BIPP Gauze®), calcium alginate (Kaltostat®) or inflatable balloon catheters (Rapid Rhino®, Foley Catheter®). Once the nose has been adequately decongested and anaesthetised with cophenylcaine (lignocaine hydrochloride plus phenylephrine), most of these packs are easy to introduce with forceps. Packs are generally left in situ for 1–5 days depending on the patient’s risk factors, coagulation state and severity of bleed.27 Consider antibiotic cover for S. aureus to minimise the risk for toxic shock syndrome.

Answer 7 Situations that would prompt you to send a patient with epistaxis to a hospital emergency department include: • signs of haemodynamic shock (eg. pallor, tachycardia, hypotension, tachypnoea, vasovagal reflex) • persistent bleeding despite adequate first aid measures • suspicion of posterior nasal bleed • recurrent anterior epistaxis not responding to cauterisation • patients with medical comorbidities requiring close observation • epistaxis secondary to severe coagulation deficits requiring reversal or blood transfusion • bleeding from a suspected nasal mass or vascular malformation • significant bleeding after nasal, sinus or anterior skull base surgery. Brand names are for example only and no particular brand is recommended or endorsed either by the editors or publishers.

24 check Ear nose and throat Case 6

case 6 Claire’s ongoing allergies Claire is a cleaner who is 32 years of age and presents with symptoms of clear nasal discharge (rhinorrhoea) and nasal irritation that have been present for the last few years but are worsening. Symptoms occur on most days but she finds them to be worse at certain times of the year, especially during the spring and summer months. Over the last 6 weeks the symptoms have been present every week. She remains otherwise well, with no drug Question 3 allergies, but reports that her daughter who is 7 years of age has had similar symptoms. What features would you be interested in on examination?

Question 1 What is the likely diagnosis?

Question 4 What investigations would you request to confirm your working diagnosis? Question 2 What features suggest this diagnosis in Claire? What other questions would you ask Claire in relation to contributory/exacerbating factors and associated conditions?

25 Case 6 check Ear nose and throat

Question 5 What would your next management step be and what therapeutic case 6 answers options are available? Answer 1 Claire presents with symptoms of rhinitis. As these are occurring on most days of the week (more than 4) and for more than 4 consecutive weeks, they are termed persistent. Previously, terminology classified rhinitis as seasonal or perennial. However, the terms intermittent and persistent are now favoured. Claire’s rhinitic symptoms may then be also subclassified as allergic or nonallergic and as mild, moderate or severe.28 From the history given, it is likely that Claire has persistent allergic rhinitis.

Answer 2 Features of Claire’s history supportive of a diagnosis of allergic rhinitis include: • the fact that her symptoms are seasonal • the fact that her daughter also has symptoms, suggesting a possible genetic predisposition • her profession as a cleaner, and therefore exposure to Question 6 chemicals in cleaning products. What recourse would you undertake if Claire’s symptoms were Taking further history should aim to help establish the initial persistent? diagnosis of allergic rhinitis and rule out alternative contributory factors such as medications. Further history should include enquiries about: • atopy – personal history of atopy including the presence of eczema and asthma and precipitation or exacerbation of symptoms with the presence of pets at home • family history of rhinitis – the fact that her daughter has similar symptoms is relevant because genetic factors within the human leukocyte antigen system have been identified in allergic rhinitic patients.29 However, the causes for allergic rhinitis are multifactorial and environmental factors play a significant part • asthma – symptoms suggestive of asthma. Take a full respiratory history and, if possible, assess airway function before and after bronchodilator use, owing to the potential of individuals with persistent rhinitis to have undiagnosed asthma or bronchial hyper-reactivity. Additionally, rhinitic patients with asthma have been shown to experience more hospital admissions and visits to GPs,30 which supports the idea of the unified airway and that rhinitis and asthma represent a continuum of disease at differing locales along the respiratory tract • occupational – Claire’s profession may also be a source of her symptoms either through exposure to an increased allergen load while cleaning or due to sensitivity to cleaning chemicals • medications – a wide range of medications including nonsteroidal anti-inflammatory drugs (NSAIDs), angiotensin coverting enzyme (commonly known as ACE) inhibitors, beta-blockers and oral contraceptives can generate nasal

26 check Ear nose and throat Case 6

symptoms. In particular, intranasal vasoconstrictors may cause • Antihistamines – the previous generation of sedating rebound rhinitic symptoms following extended use, a condition H1-blockers has given way to the second generation of termed ‘rhinitis medicamentosa’.31 nonsedating antihistamines which can provide excellent symptom control Answer 3 • Intranasal corticosteroids – intranasal corticosteroids are also Examination is secondary in the diagnostic process in this condition. excellent at controlling nasal symptoms but do not have an Examination should aim to exclude alternate pathology such as immediate onset of action. Their method of delivery and low septal deflection (septal deviation) or nasal polyposis. However, dose results in minimal risks owing to low systemic absorption features such as hypertrophied inferior turbinates and the ‘allergic but can cause dryness, bleeding and nasal irritation/stinging. salute’ which results in a crease on the nose due to the patient While effective for nasal symptoms, the presence of ocular repeatedly rubbing the nose upwards, would further lend weight to symptoms may require additional therapy directed at the eye a rhinitic aetiology. • Intranasal anticholinergics – these can be very effective in the control of rhinorrhoea and while not being a first line choice in Answer 4 allergic rhinitis, can be helpful in vasomotor rhinitis in the elderly A number of modalities exist to investigate allergic aetiologies. • Cromone – again, while effective at symptom control in a However, skin prick testing (SPT) has been shown to provide subgroup of people, their regimen requires frequent and favourable sensitivity32 when performed by trained practitioners in continued administration that can affect compliance patients who have been correctly counselled before testing, ie. to avoid medications that would suppress the wheal and flare response • Leukotriene antagonists – these have been found to be before testing. Blood tests in the form of immunoglobulin E specific, beneficial in treating symptoms of allergic rhinitis on par with 35 radioallergosorbent (RAST) tests provide lower sensitivity but higher H1-blockers but inferior to corticosteroids. specificity and do not expose the patient to the risk of anaphylaxis. When symptoms are mild, any of the above groups may be used However, the relationship between allergen identified on testing and in symptom control but when moderate or severe, intranasal symptoms is less certain with RAST testing than with SPT. corticosteroids would be preferred. A common allergy test battery would assess sensitivity to house dust mites (the most common being dermatophagoides Answer 6 pteronyssinus and farinae), grasses, pollens, fungi, cats and dogs. If symptoms persisted despite treatment, referral to a specialist should be considered. At that stage, associated anatomical Answer 5 abnormalities would be assessed and perhaps a short course of Your next management step in Claire could include advice on the oral steroids trialled. Failure of medical therapy to control symptoms following conservative measures and discussion about various initial may then prompt consideration of surgical intervention in the form medical therapeutic options. of inferior turbinoplasties to improve symptoms of nasal congestion/ obstruction. However, such intervention would be an adjunctive Conservative advice therapeutic modality as the underlying allergic aetiology would not If SPT was positive to specific allergens, allergen avoidance advice have been dealt with. Alternatively, referral to an immunologist for would be appropriate. Measures to combat dust mites include consideration of immunotherapy may be considered. However, while the reduction of soft furnishings, regular cleaning with a vacuum current regimens can provide excellent rates of control, patients cleaner which has a high efficiency particulate air filter (commonly should be counselled regarding prolonged therapeutic times, the known as HEPA filter) and the use of protective bedding. Dust mites potential for anaphylaxis and failure of treatment. thrive in areas of high humidity (>80%) and high temperature (>20°C). Therefore, closed environments using central heating may contribute to their proliferation. Humidity levels <50% cause dust mite death.33 A Cochrane Review34 has suggested that measures to reduce dust mites result in some minimal improvement in symptoms but the studies were generally small and of poor methodological stature. Broadly speaking, initial medical therapeutic options fall into the drug categories outlined below. • Decongestants – these are generally alpha-agonists and achieve symptom control through vasoconstrictive mechanisms. They should not be used for more than 3 days in a row owing to the risk of leading to rhinitis medicamentosa described in Answer 2

27 Case 7 check Ear nose and throat

Question 3 case 7 Would you arrange any investigations at this stage? Julie’s had a runny nose and feels congested Julie is 42 years of age, has a past history of asthma and is a smoker. She attends your practice with a 4 month history of postnasal drip and bilateral nasal congestion. She is otherwise well but reports being allergic to aspirin and NSAIDs.

Question 1 State the most likely diagnosis and other features within the history Question 4 that you would enquire about in relation to this. What treatment options could you offer Julie at this stage?

Question 5 What management would you consider if Julie’s symptoms failed to resolve? At what stage would you consider this?

Question 2 What would be your next step in the consultation?

Question 6 You see Julie at the 1 month mark and her symptoms are well controlled. What will you do now?

Further INFORMATION Examination of the nose reveals a reddened, swollen mucosa with no focal lesions. Julie’s sinuses are nontender. Examination of the throat and ears is normal. There are no cervical lymph nodes palpable.

28 check Ear nose and throat Case 7

Question 7 If Julie had left sided (instead of bilateral) nasal obstruction and CASE 7 ANSWERS anterior rhinoscopy revealed a polypoid lesion in her left nasal cavity, what diagnoses would you consider and what would be your next Answer 1 management step? Julie presents with symptoms that suggest chronic rhinosinusitis (CRS). Symptoms suggestive of this diagnosis that you would enquire about include nasal congestion/blockage/obstruction, anterior nasal discharge and postnasal drip. Symptoms such as halitosis and mouth breathing may occur secondary to either nasal obstruction or nasal discharge. Additionally, symptoms of facial pressure or pain or reduction or loss of sense of smell may be present. When these symptoms are present for more than 12 weeks in 1 year, they are termed chronic (persistent) rhinosinusitis and acute (intermittent) when they occur for fewer than 12 weeks in 1 year.36 The history should be completed with enquiries into the presence of nasal bleeding, crusting, cacosmia (perception of a foul odour when none exists), and unilateral symptoms. The presence of nasal bleeding, cacosmia or unilateral symptoms or the presence of neurological, visual or systemic symptoms would merit urgent referral to an ENT specialist. Question 8 Are there any other features in Julie’s history that prompt special Answer 2 consideration? The next step would be to examine Julie. Examination of the anterior portion of the nasal cavity (anterior rhinoscopy) involves inspection with the naked eye and then inspection with a headlight, nasal speculum or otoscope to identify the presence of any signs of infection such as mucopus or obstructive lesions such as a septal deflection, nasal polyposis or turbinate hypertrophy.

Answer 3 Investigations such as X-ray and CT scanning are not helpful at this presentation. X-ray and CT scans are not usually necessary when managing uncomplicated sinonasal disease in general practice Plain radiograph correlation with CRS is poor although CT can be useful in surgical planning to identify anatomical features.37 Magnetic resonance imaging is not indicated for the diagnosis or management of CRS but may be useful in combination with CT for the differentiation of fungal and neoplastic disease. Question 9 If the history and/or examination suggested sinister pathology, What red flag symptoms are important to ascertain in cases of sinus investigations targeted towards the suspected pathology should be pathology? considered.

Answer 4 If your history and examination led to the diagnosis proposed in Question 1, then treatment with a topical steroid spray together with nasal douching is appropriate. Nasal douching has potential beneficial effects through mechanical cleansing of nasal mucous and also improvement in mucociliary transport. Nasal douching can be administered using commercial preparations or may be made up by adding 1 teaspoon of salt (non-iodised with no caking ingredients) and 1 teaspoon of baking soda to a pint of water that has been boiled and cooled. This can then be sniffed in from a cupped hand to the

29 Case 7 check Ear nose and throat

postnasal space and then spat out. The side effect profile for topical Answer 8 steroid sprays is minimal and mainly includes initial stinging sensation, Three features in Julie’s history prompt special consideration. nasal dryness and epistaxis. The initial stinging sensation usually settles • First, her smoking is important. Smoking and other atmospheric after a few days of use but may be reduced through the use of aqueous pollutants have been shown to affect nasal mucosa in vitro, based intranasal corticosteroid preparations. however, population based data has not supported this40 The side effect of dryness may be helped through douching, and • Second, Julie’s asthma is important. In patients with asthma in epistaxis helped through instilling the nasal spray such that it is not coexistence with nasal polyposis, the literature presents evidence always directed medially toward the nasal septum. Systemic absorption that control of nasal polyposis through surgical measures may 38 and influence upon the hypothalamic pituitary axis are minimal owing have beneficial effects on pulmonary function41 to the relatively low concentration dose and local nature of application • Third, her aspirin sensitivity is important when combined with and should therefore not discourage prescription of topical nasal steroids. nasal polyposis and asthma. This symptom complex has been termed Samter triad. Patients with Samter triad are notoriously Answer 5 difficult to manage owing to their recalcitrant disease process If symptoms are not controlled after 1 month of the above treatment, despite medical and surgical therapy and may require frequent referral to an ENT specialist is advised. An ENT specialist will often courses of oral steroids and nasal polypectomy procedures. investigate infective components and comorbidities, including allergies, Consequently, a multidisciplinary approach including a chest and consider further trials with topical and oral steroids. Additionally, a physician, ENT surgeon, endocrinologist and immunologist may be 3 month trial of macrolide therapy may also be prescribed. The use of appropriate. macrolides in this manner is related to their anti-inflammatory properties and has been shown to have good efficacy.39 A desensitisation injection Answer 9 regimen may be appropriate where a specific allergy has been identified Red flag symptoms that are important to ask about include: on skin prick testing. Surgery may be considered in specific cases if medical therapy fails. • unilateral nasal obstruction or discharge – the differential diagnoses of unilateral symptoms have been discussed in Answer 7 Answer 6 • symptoms due to extension of pathology into the nearby orbit – If Julie’s symptoms are controlled at the 1 month mark, the treatment these symptoms include epiphoria (watery eyes), particularly if it is regimen with intranasal steroids and nasal douching should be unilateral, diplopia and reduced vision. Owing to the proximity of the continued to the 3 month mark followed by a period of cessation with orbit to the nose and paranasal sinuses, pathologies within the latter recommencement should symptoms recur. region may cause obstruction of the nasolacrimal duct resulting in tearing. Expansion may result in globe displacement with invasion, possibly causing ophthalmoplegia and/or visual loss. The coexistence Answer 7 of such findings would merit urgent specialist referral The most likely diagnosis if this were the clinical scenario is a benign inflammatory polyp. A benign inflammatory polyp is the most common • symptoms which may indicate serious underlying pathologies. These cause of persistent unilateral obstruction if anatomical deformities such include: as septal deflection are excluded. A benign inflammatory polyp can –– blood in the saliva or in the nasal discharge. This could suggest appear as a uniform, slightly translucent swelling usually arising from neoplasm beneath the middle turbinate. If large, a polyp may occlude the entire –– constitutional symptoms such as weight loss and anorexia could nasal passage and even present at the nares (nostrils). suggest neoplasm or an inflammatory disorder Hypertrophy of the inferior turbinate may appear similar to a nasal –– respiratory symptoms such as cough and shortness of breath or polyp and be misinterpreted as such. However, hypertrophied inferior recurrent respiratory infections could suggest cystic fibrosis or turbinates appear more opaque, arise from the lateral wall and are very Kartagener syndrome in the paediatric population. sensitive if touched by a probe. Further, application of a decongestant spray can reduce the turbinates to clarify their nature but will not alter the size of true nasal polyps. However, as the history in Julie’s case suggests unilateral symptoms, it is important to consider the possibility of an antrochoanal polyp, inverted papilloma, or other sinonasal inflammatory or neoplastic aetiologies. The initial presentation of unilateral signs and symptoms should always prompt caution and consequently the next management step would be to refer to a specialist to exclude sinister pathologies. A specialist is likely to perform formal nasendoscopy to assess for sinister pathologies and identify the presence of further polyps on the contralateral side.

30 check Ear nose and throat Case 8

Question 2 case 8 What is the diagnosis and what organisms can cause this condition Jim has painful blocked ears with otoscopy findings as shown in Figure 13? Jim is 35 years of age and presents with a 1 week history of bilateral otalgia and blocked ears. The last 2 nights he has been unable to sleep due to pain. He has tried cleaning his ears with cotton buds and has noticed yellowish material on the tip of the cotton bud.

Question 1 What is the likely diagnosis? What features are important to ascertain when taking Jim’s history?

Question 3 What would be the diagnosis if instead of the otoscopic findings above, you had noted that there was minimal discharge, the ear canals were erythematous and tender to touch with the speculum, you were able to see the tympanic membranes and otoscopic findings were as shown in Figure 14? What organism can cause this condition with otoscopy findings as shown in Figure 14?

Further history On otoscopic examination, you note the presence of purulent discharge and erythematous ear canals that are tender to touch with the speculum. You are unable to see the tympanic membranes. Otoscopy findings are shown in Figure 13.

Figure 13. Image of Jim’s ear. Reproduced with permission from hawkelibrary.com Figure 14. Image of ear canal. Reproduced with permission from hawkelibrary.com

31 Case 8 check Ear nose and throat

Question 4 What treatment would you advise and/or perform for the scenarios in CASE 8 ANSWERS Figure 13 and 14? Answer 1 The likely diagnosis is otitis externa. This is an infection involving the skin of the external auditory canal. In your initial history, it is important to ask Jim about recent upper respiratory tract infections as this could guide your diagnosis toward otitis media rather than otitis externa. Also, ask about predisposing factors for otitis externa including water exposure (eg. regular swimming), trauma from cotton buds or fingernails, known narrow ear canals and past history of exostoses. Ask Jim about symptoms of itchiness and skin scaling from the ear canals. This could suggest underlying skin conditions such as eczema, seborrhoeic dermatitis or psoriasis which may predispose to refractory otitis externa.42,43

Answer 2 The diagnosis is bacterial otitis externa. The common bacteria involved Question 5 are Pseudomonas aeruginosa and S. aureus.44 What advice would you give Jim to help avoid future recurrences of this condition? Answer 3 The diagnosis is otitis externa, caused by Aspergillus, a fungus. After Another fungal cause of otitis externa is Candida albicans, though this results in different otoscopic findings.44

Answer 4 The treatment involved is outlined below.

Ear canal debridement (for discharging otitis externa) Thorough ear canal debridement (ear toileting). Referral to an ENT service is often required for this. Debridement is performed using a head light and fine suction (Figure 15).

Question 6 In cases of otitis externa, what important diagnosis, although uncommon, should you have a high index of suspicion for and not miss? Describe the features on history and examination of this diagnosis.

Figure 15. Aural speculae and suction devices. Author’s image

32 check Ear nose and throat Case 8

Figure 16. Dental brooch with cotton wool. Author’s image

If suction is not available, a dental brooch (Figure 16) with a small piece of cotton wool wound around the tip can be used, under direct vision and with care. This procedure requires skill and training, as the ear is often exquisitely painful in otitis externa and trauma to the canal wall or tympanic membrane can occur with this technique. Ear toileting may need to be repeated after several days to ensure the drops are effective. It is important to debride sufficiently to visualise the tympanic membrane before coming to a definitive diagnosis of otitis externa. Otitis media with a discharging perforation may mimic otitis externa.45 Figure 17. Otowicks. Author’s image

Ear drops Bacterial otitis externa can be treated with either framycetin sulphate/gramicidin/dexamethasone ear drops or ciproxin hydrocortisone ear drops. Treatment is usually continued for 7–10 days. If there is concern about a perforated tympanic membrane, ciproxin drops should be used without the steroid component, which may cause stinging. An otowick is inserted (Figure 17 and 18) if there is marked canal oedema. The wick helps to reduce oedema, delivering the topical antibiotic and steroid preparation deep in the canal. It is normally removed after 48 hours and further ear toileting performed. Fungal otitis externa may be treated with either flumethasone pivalate/cliquinol drops for 2 weeks or alternatively, the ear canal may Figure 18. An otowick inserted into ear canal. Author’s image be filled with otocomb ointment for 1 week, followed by flumethasone pivalate/cliquinol for 1 week. an over-the-counter preparation containing alcohol and acetic acid after swimming. Use of cotton buds should be avoided. Any underlying skin Oral analgesia conditions should be treated. Moderately strong analgesia may be required for several days.

Treatment of underlying skin conditions Answer 6 Underlying skin conditions, such as eczema and psoriasis, should be A diagnosis that is important not to miss is malignant otitis externa (or treated. skull base osteitis). Malignant otitis externa is a potentially life threatening infection of the external ear and skull base. This most commonly presents Feedback in elderly patients, diabetic patients and the immunocompromised (eg. HIV-infected patients and postchemotherapy patients). It usually Oral antibiotics are not normally required unless there is associated presents with unrelenting otalgia over several weeks. On examination perichondritis (in which case a course of oral ciprofloxacin is given), there is ear discharge, with granulation tissue visible in the floor of the concurrent otitis media or systemic illness.46 external ear canal. There may be facial nerve or other lower cranial nerve Microbiologic swabs are not normally necessary to direct treatment involvement. Investigations include: unless the patient is not responding to the above treatments or • CT scan of the temporal bones to document the extent of disease malignant otitis externa is suspected. • bone scan to document osteomyelitis Answer 5 • gallium scan to document and track active disease. Jim should keep his ears dry for 2 weeks after this infection has Malignant otitis externa is treated with a prolonged course of intravenous resolved. When showering Jim should use cotton wool coated in and oral antibiotics in consultation with an infectious disease unit. A high petroleum jelly to prevent water entering his ears. If he is a swimmer, index of suspicion is needed for early diagnosis in high risk populations as he could consider custom made ear plugs for swimming and the use of the condition has a significant mortality.42 Urgent referral is indicated.

33 resources and References check Ear nose and throat

resources 21. Massick D, Tobin EJ. Epistaxis. In: Cummings CW, Flint PW, Haughey BH, et al, • Hawke Library of Otolaryngology. Available at www.hawkelibrary.com editors. Otolaryngology head and neck surgery. Philadelphia: Elsevier Mosby, 2005. 22. Calder N, Kang S, Fraser L, et al. A double-blind randomized controlled trial of • Benign paroxysmal positional vertigo management of recurrent nosebleeds in children. Otolaryngol Head Neck Surg (nystagmus during Dix Hallpike Manoeuvre. Available at http://video. 2009;140:670–4. google.com/videoplay?docid=-6201505057633350143#) 23. Douglas R, Wormald PJ. Update on epistaxis. Curr Opin Otolaryngol Head Neck • Epley manoeuvre video. Available at www.australianprescriber.com/ Surg 2007;15:180–3. magazine/28/4/94/7 24. Chiu TW, McGarry CW. Prospective study of bleeding sites in idiopathic adult posterior epistaxis. Otolaryngol Head and Neck Surg 2007;137:390–3. • Video of head impulse test in unilateral vestibular loss. Available at www. 25. Supriya M, Shakeel M, Veitch D, et al. Epistaxis: prospective evaluation of bleeding youtube.com/watch?v=P68SVfl7vAk&feature=player_detailpage#t=38s. site and its impact on patient outcome. J Laryngol Otol 2010;124:744–9. 26. Gifford TO, Orlandi RR. Epistaxis. Otolaryngol Clin N Am 2008;41:525–36. References 27. Daudia A, Jaiswal V, Jones NS. Guidelines for the management of idiopathic 1. Ruotsalainen JH, Sellman J, Lehto L, et al. Interventions for treating functional epistaxis in adults: how we do it. Clin Otolaryngol 2008;33:618–20. dysphonia in adults. Cochrane Database Syst Rev 2007;3:CD006373. 28. Bousquet J, Khaltaev N, Cruz AA, et al. Allergic rhinitis and its impact on asthma 2. Reveiz L, Cardona AF, Ospina EG. Antibiotics for acute laryngitis in adults. (ARIA): 2008 update (in collaboration with the World Health Organization, GA(2) Cochrane Database Syst Rev 2007;2:CD004783. LEjdwdN and AllerGen). Allergy 2008;63(Suppl 86):8–160. 3. Schwartz SR, Dailey SH, Deutsch ES, et al. Clinical practice guideline: 29. Ciprandi G, Contini P, Murdaca G, et al. Soluble HLA-G molecule in patients with hoarseness (dysphonia). Otolaryngology-head and neck surgery 2009;141:S1– perennial allergic rhinitis. Int Arch Allergy Immunol 2009;150:278–81. S31. 30. Bousquet J, Gaugris S, Kocevar VS, et al. Increased risk of asthma attacks and 4. Wilson WR, Byl FM, Laird N. The efficacy of steroids in the treatment of emergency visits among asthma patients with allergic rhinitis: a subgroup analysis idiopathic sudden hearing loss: a double-blind clinical study. Arch Otolaryngol of the investigation of montelukast as a partner agent for complementary therapy 1980;106:772. [corrected]. Clin Exp Allergy 2005;35:723–7. 5. Pichichero ME, Poole MD. Comparison of performance by otolaryngologists, 31. Graf P. Rhinitis medicamentosa: a review of causes and treatment. Treat Respir pediatricians, and general practitioners on an otoendoscopic diagnostic video Med 2005;4:21–9. examination. Int J Pediatr Otorhinolaryngol 2005;69:361–6. 32. Choi IS, Koh YI, Koh JS, et al. Sensitivity of the skin prick test and specificity of the 6. Bluestone CD, Casselbrant ML, Dohar JE. Targeting therapies in otitis media and serum-specific IgE test for airway responsiveness to house dust mites in asthma. J otitis externa. Ontario: BC Decker, 2003. Asthma 2005;42:197–202. 7. Bluestone CD. Eustachian tube structure, function, role in otitis media. Ontario: 33. Lintner TJ, Brame KA. The effects of season, climate, and air-conditioning on the BC Decker, 2005. prevalence of dermatophagoides mite allergens in household dust. J Allergy Clin Immunol 1993;91:862–7. 8. Foxlee R, Johansson AC, Wejfalk J, et al. Topical analgesia for acute otitis media. Cochrane Database Syst Rev 2006;3:CD005657. 34. Sheikh A, Hurwitz B, Nurmatov U, van Schayck CP. House dust mite avoidance measures for perennial allergic rhinitis. Cochrane Database Syst Rev. 9. Sanders S, Glasziou PP, Del Mar CB, et al. Antibiotics for acute otitis media in 2010;7:CD001563. children. Cochrane Database Syst Rev 2004;1:CD000219. 35. Rodrigo GJ, Yanez A. The role of antileukotriene therapy in seasonal allergic 10. Antibiotics Expert Group. Therapeutic guidelines: antibiotic. Version 13. rhinitis: a systematic review of randomized trials. Ann Allergy Asthma Immunol Melbourne: Therapeutic Guidelines limited, 2006. 2006;96:779–86. 11. Coleman C, Moore M. Decongestants and antihistamines for acute otitis media 36. Fokkens W, Lund V, Mullol J. European position paper on rhinosinusitis and nasal in children. Cochrane Database Syst Rev 2008;3:CD001727. polyps 2007. Rhinol Suppl 2007;20:1–136. 12. American Academy of Family Physicians. American Academy of Otolaryngology- 37. Iinuma T, Hirota Y, Kase Y. Radio-opacity of the paranasal sinuses. Conventional Head and Neck Surgery and American Academy of Pediatrics Subcommittee on views and CT. Rhinology 1994;32:134–6. Otitis Media With Effusion. Pediatrics 2004;113:1412–1429. 38. Skoner D. Update of growth effects of inhaled and intranasal corticosteroids. Curr 13. Leach A, Morris PS. Antibiotics for the prevention of acute and chronic Opin Allergy Clin Immunol 2002;2:7–10. suppurative otitis media in children. Cochrane Database Syst Rev 2006;4:CD004401. 39. Ichimura K, Shimazaki Y, Ishibashi T, et al. Effect of new macrolide roxithromycin upon nasal polyps associated with chronic sinusitis. Auris Nasus Larynx 14. Cnyrim CD, Newman-Toker D, Karch C, et al. Bedside differentiation of vestibular 1996;23:48–56. neuritis from central ‘vestibular pseudoneuritis’. J Neurol Neurosurg 2008;79:458–60. 40. Bousquet PJ, Cropet C, Klossek JM, et al. Effect of smoking on symptoms of allergic rhinitis. Ann Allergy Asthma Immunol 2009;103:195–200. 15. Sajjadi H, Paparella M. Meniere’s disease. Lancet 2008;372:406–14. 41. Batra PS, Kern RC, Tripathi A, et al. Outcome analysis of endoscopic sinus surgery 16. Committee on Hearing Equilibrium. Guidelines for the diagnosis and evaluation in patients with nasal polyps and asthma. Laryngoscope 2003;113:1703–6. of therapy Meniere’s disease. Amer Acad Otolaryngol Head Neck Surg Foundation Inc. Otolaryngol Head Neck Surg 1995;113:176–78. 42. Lee KJ. Essential otolaryngology. 9th edn. New York: Mc Graw-Hill, 2008. 17. Neuhauser H, Leopold M, von Brevern M, et al. The interrelations of migraine, 43. Ludman H, Bradley P. ABC of ear, nose and throat. 5th edn. Malden, Mass: vertigo and migrainous vertigo. Neurology 2001;56:436–41. Blackwell Publishing. 18. Neurology Expert Group. Motion sickness and vertigo. In: Therapeutic Guidelines: 44. Brown CL, van der Straaten D, Joyce L, et al. Acute otitis externa – a local experience neurology. Version 3. Melbourne: Therapeutic Guidelines Limited, 2007;89–101. of clinical features and microbiology. Aust J Otolaryngology 2002;5:90–4. 19. Strupp M, Carina Zingler V, Arbusow V, et al. Methylprednsiolone, valacyclovir or 45. Bull B, Clarke R. Lecture notes diseases of the ear, nose and throat. 10th edn. the combination for vestibular neuritis. New Engl J Med 2004;351:354–61. London: Blackwell Publishing, 2007. 20. Bronstein A, Lempert T. Dizziness: a practical approach to diagnosis and 46. Pasha, R. Otolaryngology head and neck surgery clinical reference guide. 2nd edn. management (Cambridge Clinical Guides). Cambridge: Cambridge University San Diego: Plural Publishing, 2006. Press, 2007.

34 check Ear nose and throat check Category 2 QI&CPD activity

it lateralises to the left ear. Otoscopic examination of the left ear reveals a dull Ear nose and throat tympanic membrane without erythema. Which of the following is the most In order to qualify for 6 Category 2 points for the QI&CPD likely cause for his deafness? activity associated with this unit: A. Acute otitis media • read and complete the unit of check in hardcopy or B. Otitis externa online at the gplearning website at www.gplearning. C. Idiopathic sudden sensorineural hearing loss com.au, and D. Occupational deafness • log onto the gplearning website at www.gplearning. com.au and answer the following 10 multiple choice E. Otitis media with effusion (middle ear effusion). questions (MCQs) online. • complete the online evaluation. Question 3 Jan is 66 years of age with a past history of diabetes which is well controlled If you are not an RACGP member, please contact the on metformin. She also takes simvastatin and rampiril. When she got out of gplearning helpdesk on 1800 284 789 to register in the bed a few days ago she noticed that she was ‘off balance’, which settled first instance. You will be provided with a username and quickly, and has been recurring intermittently since, each time lasting a few password that will allow you access to the test. minutes. She has no nausea or vomiting and no deafness or tinnitus. Her The expected time to complete this activity is 3 hours. blood sugar levels have been between 6 and 8. Examination reveals a well Please note: looking lady who is afebrile, pulse rate is 76 and in sinus rhythm, blood • from January 2011, there will no longer be a Category 1 pressure 135/80 standing and 140/75 lying. Otoscopic examination and activity (ALM) associated with check units. This decision cardiovascular examination are normal and there are no neurological deficits. was made due to a lack of interest in this activity. The The Hallpike manoeuvre is positive. Which of the following is the most likely RACGP apologises for any inconvenience caused by this cause for her symptoms? change A. Vestibular neuritis • do not send answers to the MCQs into the check office. B. Benign paroxysmal positional vertigo This activity can only be completed online at www. C. Hypoglycaemia gplearning.com.au. D. Hypotension If you have any queries or technical issues accessing the E. Viral illness. test online, please contact the gplearning helpdesk on 1800 284 789. Question 4 Susan is 59 years of age with a past history of migraine who presents to your Question 1 practice at lunchtime with a fairly sudden onset of a throbbing headache, vertigo, nausea and vomiting which started earlier in the day. The headache Chloe is 26 years of age, she is a physical education teacher who resembles her usual migraine in character but is more severe and failed noticed a sore throat and runny nose 2 weeks ago and then developed to respond to a single dose of sumatriptan as it usually does. You note a productive cough and hoarseness of voice soon after. She has had that she staggers into your room and has nystagmus during history taking. no fever and has not felt unwell. She is frustrated by her hoarseness of Examination reveals normal otoscopic findings, an intention tremor (but voice, which she notices is worse in the afternoon after coaching the no other neurological signs), a negative Romberg test and a normal head school sports teams all day. The most likely cause for her hoarseness of impulse test. Susan declines testing with the Hallpike manoeuvre unless it is voice is: absolutely necessary. The next most appropriate step in the management of A. viral laryngitis Susan is to: B. bacterial laryngitis A. administer another dose of sumatriptan C. muscle tension dysphonia B. prescribe oral prochlorperazine maleate and an oral codeine-based D. postnasal drip analgesic E. vocal nodules. C. administer a parenteral antiemetic such as intramuscular prochlorperazine maleate and a parenteral analgesic such as tramadol Question 2 D. arrange referral to hospital Samuel is 26 years of age and works in a factory. He presents to you E. explain the importance of performing the Hallpike manoeuvre and after having noticed deafness in his left ear when he woke up this morning, obtaining consent, perform the Hallpike manoeuvre to determine the likely following his return the night before from an interstate holiday. He has a cause of her symptoms. longstanding history of hay fever and has noticed occasional ‘popping’ of his ears. He has no ear pain and no fever. You perform a Weber test and

35 check Category 2 QI&CPD activity check Ear nose and throat

Question 5 C. topical antibiotic/antiseptic ointments Stephanie is 4 years of age and presents the morning after the onset of D. topical intranasal steriods a right sided earache. Stephanie and her family are well known to you E. air humidifers. and Stephanie has had an occasional episode of otitis media in the past and has no allergies. She is quieter than usual but is cooperative. Her Question 8 temperature is 37.4°C per aural, and otoscopic examination of her right ear reveals an erythematous, bulging drum. Management in line with Tim is 55 years of age and is an office worker who migrated to Australia current recommendations could include: 4 years ago. For each of the last 3 years he has experienced sneezing, itchiness of the nose and clear nasal discharge. The symptoms start in the A. advising use of paracetamol in the appropriate dose and intervals and spring and continue for several months and have been getting progressively review in 5 days worse each year. He has a past history of asthma, for which he uses B. prescribing amoxycillin and advising her mother to fill the script occasional salbutamol. He has no other past history. He is frustrated as his straight away nasal symptoms are getting progressively worse each year. This year he has C. prescribing amoxycillin and advising her mother to fill the script if tried intranasal corticosteroids for 6 weeks without effect. You examine him Stephanie is still in pain in 48 hours and note nasal inflammation is present. All of the following are appropriate in D. prescribing amoxycillin/clavulanate and advising her mother to fill the the management of Tim’s nasal symptoms except: script straight away A. referral for skin prick testing E. prescribing amoxycillin/clavulanate and advising her mother to fill the B. advice on allergen avoidance script if Stephanie is still in pain in 48 hours. C. use of a 2 week trial of oral antihistamines D. use of a 2 week trial of intranasal decongestants Question 6 E. enquiry about environmental triggers. Ella is age 15 months and presents to your practice for review following an episode of acute otitis media with perforation for which you Question 9 prescribed a course of amoxycillin. Her growth and development have been normal and she is fully immunised. You ascertain that her infection Leanne is 40 years of age and presents for review 1 month following an has now cleared and her perforation has healed. It is her fourth episode episode of acute sinusitis for which you prescribed a course of amoxycillin of otitis media in the last 6 months. Her mother Tracy says she has been as she had failed to respond to decongestants. Her facial pain and purulent talking with some of the other mothers at the childcare centre that Ella nasal discharge settled with the course of antibiotics, but for the last has been attending for the last 6 months who have recommended that 3 weeks she has been troubled by a postnasal drip and feels the need to she speak with you about the possibility of Ella having ventilation tubes continually clear her throat. She has had no other symptoms. Examination is inserted. Which of the following is true regarding recurrent otitis media? normal. The next most appropriate step in her management would be: A. Prophylactic cefaclor is recommended to reduce the risk of further A. referral to an ear, nose and throat (ENT) surgeon episodes of otitis media B. request a computed tomography scan of her sinuses to check for the B. Insertion of ventilation tubes reduces the risk of chronic otitis media presence of polyps with effusion but not the risk of recurrent otitis media C. advise use of an intranasal steroid spray and nasal douching C. Audiology testing is often performed before ventilation tubes are D. referral to an allergist to institute a desensitisation regimen inserted in cases where there is concern about hearing E. arrange for skin prick testing. D. An underlying significant cause such as immunodeficiency or cystic fibrosis is likely to be operating in Ella’s case Question 10 E. Reassurance that, in cases such as Ella’s, there is no need for Aidan is 22 years of age and is a surfer who presents to your practice with a ventilation tubes or any further follow up. painful left ear of 1 week duration. He has noticed yellow discharge coming from the left ear for the last 2 days. He has also had significant ‘stuffiness’ Question 7 of his nose. Otoscopic examination of his left ear is very painful and reveals Nelson is 55 years of age and presents with a history of four nose copious yellow discharge but you are unable to visualise his ear drum. The bleeds in the last 2 weeks. Each nose bleed has lasted about 15–20 most appropriate next step in management would be to: minutes. The last episode was a couple of days ago. Nelson is otherwise A. prescribe benzocaine ear drops well with no relevant past history. He is not on any medications. You B. prescribe sulphate/gramicidin/dexamethasone ear drops examine his nose internally and notice a clot of blood present superiorly. C. prescribe ciproxin ear drops All of the following are appropriate in the management of Nelson except: D. advise Aidan to use cotton buds to remove the discharge A. advice on first aid should his epistaxis recur E. remove the discharge and debris if you are appropriately skilled, or B. consideration of cauterisation arrange for an ENT specialist to do so, so that the eardrum can be visualised.

36 GP11 THE CONFERENCE FOR GENERAL PRACTICE

Call for abstracts extended

The GP11 committee invites abstract submissions for the following key themes: • Women’s and children’s medicine • Aboriginal and • Innovations in education Torres Strait Islander health • Telemedicine • Business of medicine • Clinician leadership development • Dermatology in practice • Informatics / e-health • Emergency medicine

Submissions close 15 April 2011

www.gp11.com.au

Hotel Grand Chancellor Hobart 6 – 8 October 2011

GP11_Check_ad_April_2011.indd 1 10/03/2011 11:54:52 AM THE RACGP PRESENTS A NEW LEVEL OF CLINICAL INTELLIGENCE

2011 New functionality available soon

To register your interest email [email protected]

Sidebar_GP_ad_275mmx355mm REVISED.indd 1 11/03/11 1:56 PM